Nell participated in a 3-day charity walk. Sheraised $0.50 for each 1/3 of mile that she walked. Thefirst day, Nell walked 12 miles. The second day, shewalked 8 miles. The third day, she walked 16 miles.How much money did Nell raise?

Answers

Answer 1

the Numbers of days = 3

The second statement(She raised $0.50 for each 1/3 of mile that she walked) means

1/3 miles = $0.50

For first day walk we have:

1/3 miles = $0.50

12 miles = 12 x 0.50 x 3 = $18

For Second day walk we have:

1/3 miles = $0.50

8miles = 8 x 0.50 x 3 = $12

For Third day walk we have:

1/3 miles = $0.50

16miles = 16 x 0.50 x 3 = $24

The amount she raised for the three days becomes

$18 + $12 + $24

=$54


Related Questions

solveeeeeeeeeeeee for x

Answers

Answer:

7.5/3

Step-by-step explanation:

set AC and BD equal to eachother because they are equal and solve

Use the elimination method to solve the system of equations.3x + 4y = 8x - y = 12

Answers

Answer:

We have to use to elimination method to solve the provided system of equations, this method essentially involves eliminating one of the variables in the first equation by adding or subtracting a multiple of the second equation:

[tex]\begin{gathered} 3x+4y=8\rightarrow(1) \\ \\ x-y=12\rightarrow(2) \end{gathered}[/tex]

The elimination of the variable y in equation (1) is achieved by adding the 4 times the equation 2 into the equation (1), the steps of the process are as follows:

[tex]\begin{gathered} 3x+4y=8 \\ \\ + \\ \\ 4\times(x-y)=4\times12 \\ \\ ------------------------------- \\ \\ 3x+4x=8+48 \\ \\ \\ 7x=56 \\ \\ \\ x=8 \end{gathered}[/tex]

We have found the x value, the last step is to plug in the variable x value in any of the (1) or (2) equations, and solve for the y variable, the steps are as follows:

[tex]\begin{gathered} x-y=12 \\ \\ x=8 \\ \\ \therefore\Rightarrow \\ \\ 8-y=12\rightarrow y=8-12=-4 \\ \\ y=-4 \end{gathered}[/tex]

The final answer is as follows:

[tex](8,-4)\Rightarrow\text{ Option \lparen B\rparen}[/tex]

Therefore the answer is Option(B).

Someone please Help!! 30 Points

There are currently 1,250 pine trees on Judy's plot of land. The number of pine trees on her land is currently decreasing at a rate of 14% per year.
Which of the following graphs shows the number of pine trees, Np, on Judy's land x years from now?

Answers

Answer: we would need to see the graphs to address this question fully, but you should look for the graph that starts at x = 0, 1250 and decreases by 14% compared to the previous year

so for example, x = 1, the y value should be 1250 - (0.14)(1250)

Step-by-step explanation:


BD bisects ABC, m/ABD = 2(3x), and m/ABC = 42°.
What is the value of x?

Answers

In the given triangle ABC where BD bisects ABC and according to the values of the given angles, the value of x is found out to be 3.5.

It is given to us that -

BD bisects ABC.

m/ABD = 2(3x) ----- (1)

and, m/ABC = 42 ------- (2)

We need to find out the value of x.

Since BD bisects triangle ABC,

The angles created with respect to the bisector are equal.

=> m/ABD = m/CBD ----- (3)

Also, it is also known that -

m/ABD + m/CBD = m/ABC ----- (4)

Substituting equation (3) in equation (4),

m/ABD + m/CBD = m/ABC

=> m/ABD + m/ABD = m/ABC

=> 2 m/ABD = m/ABC ----- (5)

Substituting the values of m/ABD and m/ABC from equation (1) and (2) in equation (5), we get

2 m/ABD = m/ABC

=> 2*2(3x) = 42

=> 12x = 42

=> x = 3.5

Thus, according to the values of the angles of the triangle, the value of x is 3.5.

To learn more about triangles visit

https://brainly.com/question/2773823

#SPJ9

Circle O has a center of (-1,5) and passes through the point (2,9) write the equation in center radius form and standard form

Answers

Firstly, calculate the radius r of the circle using distance between two points

[tex]\begin{gathered} r\text{ = }\sqrt[]{(2-(-1))2+(9-5)^2} \\ r=\text{ }\sqrt[]{(2+1)^2+(4)^2} \\ r=\sqrt[]{3^2+4^2} \\ r=\sqrt[]{9+16} \\ r=\sqrt[]{25} \\ r=\text{ 5} \end{gathered}[/tex]

Then use the radius r=5 and the center (-1 , 5) to find the equation of the circle

[tex]\begin{gathered} r^2=(x-h)^2+(y-k)^2 \\ 5^2=(x-(-1))^2+(y-5)^2 \\ 5^2=(x+1)^2+(y-5)^2 \end{gathered}[/tex]

The center radius form is

[tex]5^2=(x+1)^2+(y-5)^2[/tex]

The standard form is

[tex]\text{ 25}^{}=(x+1)^2+(y-5)^2[/tex]

Mr. March sells popcorn at his theater. He uses 3 3/4 cups of unpopped corn to make 15 bags of popped corn. Write an equation for the number of bags of popcorn b that can be made with c cups of unpopped corn.​

Answers

The equation for the number of bags of popcorn 'b' that can be made with 'c' cups of unpopped corn is

b = 4c.

Given, Mr. March sells popcorn at his theater.

He uses 3(3/4) cups of unpopped corn to make 15 bags of popped corn. We have to write an equation for the number of bags of popcorn 'b' that can be made with 'c' cups of unpopped corn.​

Now, according to the question

3(3/4) cups = 15 bags

15/4 cups = 15 bags

1 cup = 4 bags

So, in 'c' cups he will make, 4c bags

therefore, 4c = b

Hence, the equation for the number of bags of popcorn 'b' that can be made with 'c' cups of unpopped corn is

b = 4c.

Learn more about Linear Equations here https://brainly.com/question/24085666

#SPJ9

the scatter plot below was constructed using data on length and inches (x) of several alligators and weight in pounds (y)right if you sent this describing the relationship between weight and length for these alligators

Answers

Answer

We are told that the relationship describing the weight and length, for alligators.

The length in inches is presented on the x-axis and the weight in pounds is presented on the y-axis.

The observations about this include

- From the scatterplot, we can see that the more the length of the alligator (that is, as x increases), the weight of the alligator also increases (that is, y increases too).

- This points to a direct relationship between the length of alligator and the weight of the alligator.

- Small alligators weigh light weights and longer alligators are heavy.

Hope this Helps!!!

Charlotte states that (43)3
can be rewritten as 218
. Which of the following explains how she is correct? Select all that apply.

Answers

The statement that explains why Charlotte's submission is correct is that (4³)³ simplifies to (4⁹) and (4⁹) simplifies to 2¹⁸

How to determine why Charlotte's submission is correct?

From the question, the expression is given as

(43)3

Rewrite the expression properly

So, we have the following representation

(4³)³

Apply the law of indices to rewrite the exponents in the expression properly

So, we have

(4³)³ = 4³*³

Next, we evaluate the products

So, we have the following notation

(4³)³ = 4⁹

Solving further, we need to express 4 as the square of 2

So, we have

(4³)³ = (2²)⁹

Apply the law of indices to rewrite the exponents in the expression properly

So, we have

(4³)³ = 2²*⁹

Next, we evaluate the products

So, we have the following notation

(4³)³ = 2¹⁸

Hence, Charlotte's expression is correct

Read more about expressions at

https://brainly.com/question/4344214

#SPJ1

Possible question

Charlotte states that (4³)³ can be rewritten as 2¹⁸. Which of the following explains how she is correct? Select all that apply.

How do you divide x by a fraction, or an example of dividing x by a fraction.

Answers

SOLUTION

Now this is how I can divide x by a fraction.

Let's divide x by

[tex]\frac{2}{3}[/tex]

So

[tex]x\text{ divided by }\frac{2}{3}[/tex]

Now to do this, change division to multiplication. When you do this, the fraction reverses. That is the number up will come down and the number down will go up.

So, this becomes

[tex]\begin{gathered} x\text{ divided by }\frac{2}{3} \\ x\times\frac{3}{2} \end{gathered}[/tex]

x on its own means x over 1, that is

[tex]\begin{gathered} x\times\frac{3}{2} \\ \frac{x}{1}\times\frac{3}{2} \\ m\text{ultiplying we have } \\ \frac{3x}{2} \end{gathered}[/tex]

So this is how to divide x by a fraction.

Type the correct answer in each box. Use numerals instead of words. This graph represents a quadratic function. What is the function’s equation written in factored form and in vertex form? Graph shows upward parabola plotted on a coordinate plane. The parabola has vertex at (2, minus 8) with the left slope at (0, 0) and the right slope at (4, 0).

Answers

The function’s equation written in factored form and in vertex form are respectively;

f(x) = 2x(x - 4)

f(x) = 2(x - 2)² - 8

How to Interpret Quadratic Graphs?

The factored form of a quadratic function is;

f(x) = a(x - p)(x - q)

where:

p and q are the x-intercepts

a is a constant

Now, we are given x-intercepts as; (0, 0) and (4, 0)

Thus;

f(x) = a(x - 0)(x - 4)

f(x) = ax(x - 4)

To find a, substitute the given vertex (2, -8) into the equation and solve for a:

2a(2 - 4) = -8

-4a = -8

a = 2

Thus;

f(x) = 2x(x - 4)

The vertex form of a quadratic equation is;

f(x) = a(x - h)² + k

where:

(h, k) is the vertex

a is constant

Since vertex is (2, -8), then we have;

f(x) = a(x - 2)² - 8

Put the coordinate (0, 0) to find a;

0 = a(0 - 2)² - 8

4a = 8

a = 2

Thus;

f(x) = 2(x - 2)² - 8

Read more about Quadratic Graphs at; https://brainly.com/question/14477557

#SPJ1

Question 3 of 30
sin 0=/941 and cos 0 =-40/41. Find tan 0.
O A. 40/9
OB. 9/40
C. -40/9
D. - 9/40

Answers

Answer:

answer Is -9/40

Step-by-step explanation:

the formula Is

tan0= sin0/cos0

tan0= (9/41)/(-40/41)

tan0=(9/41)*(-41/40)

tan0= -9/40

while shopping for clothes, Daniel spent $26 less than 2 times what curtis spent. Daniel spent $10. write and solve an equation to find how much curtis spent. let x represent how much curtis spent

Answers

while shopping for clothes, Daniel spent $26 less than 2 times what curtis spent. Daniel spent $10. write and solve an equation to find how much curtis spent. let x represent how much curtis spent​

Let

x ------> amount that Curtis spent

we have that

10=2x-26 ------> equation that represent this situation

solve for x

2x=10+26

2x=36

x=$18

therefore

Curtis spent $18

Madeline earns $17,500 annually. What is the gross amount of hersemimonthly paycheck?a. $2,916.67b. $1,458.33c. $729.17d. $673.08

Answers

Given:

Annual earnings = $17,500

Asked: What is the gross amount of her semimonthly paycheck?

Solution:

To find the answer, we need to divide first the annual earnings by 12 to get the monthly paycheck. After that, we will divide it by 2 because we are looking for the semimonthly paycheck.

[tex]\begin{gathered} \frac{17,500}{12}=1,458.33 \\ \frac{1,458.33}{2}=729..17 \end{gathered}[/tex]

ANSWER: C. $729.17

A driver accelerates her car from rest to a velocity of 35.9 mi/hr in 8.2 seconds. What is the acceleration of the car?

Answers

The acceleration of the car will be 0.002 m / s².

What is Division method?

Division method is used to distributing a group of things into equal parts. Division is just opposite of multiplications.

For example, dividing 20 by 2 means splitting 20 into 2 equal groups of 10.

Given that;

The velocity of the car = 35.9 m/hr

The time take by car = 8.2 seconds

Now,

Since, We know that;

Acceleration = (Final velocity - Initial velocity) / Time

Since, The driver accelerates her car from rest to a velocity of 35.9 mi/hr.

So, Initial velocity = 0

And, Final velocity = 35.9 m / hr

                             = (35.9 / 3600) m/s

                             = 0.01 m / s

So, We get;

⇒ Acceleration = (Final velocity - Initial velocity) / Time

⇒ Acceleration = (0.01 - 0) / 8.2

⇒ Acceleration = 0.01 / 8.2

⇒ Acceleration = 0.002 m / s²

Thus, The acceleration of the car will be 0.002 m / s².

Learn more about the divide visit:

https://brainly.com/question/25018554

#SPJ1

The area of circular pound is 21.5 m2. Work out the diameter of the circle. Give your answer correct to the nearest centimetre.

Answers

The diameter of the circle is 5.2m.

The area of circular pond is 21.5 m2

area of a circle is given by the formula

area = πr²

21.5 = 3.14 r²

r² = 6.84

r = ±2.6

r is a length and it cannot be negative

So, r = 2.6

Diameter is twice of radius

d = 2r

d = 2(2.6)

d = 5.2

Therefore, the diameter of the circle is 5.2m.

To learn more about circle refer here

https://brainly.com/question/24375372

#SPJ1

Lina picks a 4 digit number.
The number is more than 5000.
The number is odd.
The second digit is a prime number.

How many different possible numbers could Lina pick?

Answers

Answer:

1000

Step-by-step explanation:

The easiest way to go about this is by turning our number into ABCD.

A will be in the 1000th spot, B 100th, C 10th, and D 1st.

A has to be ≥ 5. So 5, 6, 7, 8, 9.

B has to be a prime number. 2, 3, 5, 7.

C is able to be any number it wants, 0, 1, 2, 3, 4, 5, 6, 7, 8, 9.

Since the number is odd, D has to be 1, 3, 5, 7, 9.

A has 5 different possibilities. B has 4, C has 10, and D has 5.
You'll then multiply all of these numbers for your answer.
5 * 4 * 10 * 5 = 1000

help meeeeeeeeeeeeeeeeeeee pleaseee rnnnn rn!!!!

Answers

The length of short side x is 4.5 units, the length of short side x+5 is 9.5 units and the length of longest side is 10.5 units.

By Pythagorean theorem,

[tex]( Hypotenuse)^{2} = (Base)^{2}+ (Perpendicular)^{2}[/tex]

Let                base = x+5

     perpendicular = x

        Hypotenuse = x+6

                                    [tex](x+6)^{2} = x^{2} +(x+5)^{2}[/tex]

                           [tex]x^{2} +12x+36 = x^{2} +x^{2}+10x+25[/tex][tex]2x^{2} +10x+25-x^{2} -12x-36=0[/tex]

                             [tex]x^{2} -2x-11=0[/tex]

                                               [tex]x = \frac{-(-2) + \sqrt{4 - 4(1)(-11)} }{2*1}[/tex] or [tex]\frac{-(-2) - \sqrt{4 - 4(1)(-11)} }{2*1}[/tex]

                                               [tex]x=\frac{2 + \sqrt{4 +44} }{2}[/tex] or  [tex]\frac{2 - \sqrt{4 +44} }{2}[/tex]

                                               [tex]x = \frac{2 + \sqrt{48} }{2}[/tex] or [tex]\frac{2 - \sqrt{48} }{2}[/tex]

                                               [tex]x = \frac{2+2\sqrt{12} }{2}[/tex] or [tex]\frac{2-2\sqrt{12} }{2}[/tex]

                                               [tex]x = 1+\sqrt{12}[/tex] or [tex]1-\sqrt{12}[/tex]

                                               [tex]x = 1+3.5[/tex] or [tex]1-3.5[/tex]

                                               [tex]x = 4.5[/tex] or [tex]-2.5[/tex]

As, length can't be negative, we will take x = 4.5

Therefore, x = 4.5

            x + 5 = 4.5 + 5

                      = 9.5

             x + 6 = 4.5 + 6

                       = 10.5

Hence, the length of short side x is 4.5 units, the length of short side x+5 is 9.5 units and the length of longest side is 10.5 units.

Learn more about length on:

https://brainly.com/question/8552546

#SPJ1

Manuel took a total of 20 quizzes over the course of 2 weeks. How many weeks of school will Manuel have to attend this quarter before he will have taken a total of 90 quizzes? Assume the relationship is directly proportional.​

Answers

Manuel will attend the quarter in 9 weeks.

Given,

In the question:

Total taken quizzes is 90

Manuel took a total of 20 quizzes over the course of 2 weeks.

To find the how many weeks of school will Manuel have to attend this quarter ?

Now, According to the question:
Based on the given conditions :

Formulate:

Let the no. of weeks be x

Total quizzes = 90

Here, The relationship is directly proportional.​

20/2 = 90/x

If the fraction is defined, the denominator cannot be equal 0.

Solve the equation:

10 = 90/x

10x = 90

x = 9

Hence, Manuel will attend the quarter in 9 weeks.

Learn more about Proportional at:

https://brainly.com/question/12235587

#SPJ1

Pls do this for me asap

Answers

a) The area of the rectangle has the following inequality: 12 ≤ 6 · x ≤ 36.

b) The value of the variable x has the following inequality: 2 ≤ x ≤ 6.

How to build and interpret an inequality associated to the area of an rectangle

The area of a rectangle (A) is described by the following formula:

A = b · h

Where:

b - Base of the rectangleh - Height of the rectangle

a) If we know that b = 2 · x and h = 3, then the area of the rectangle is:

A = (2 · x) · 3

A = 6 · x

And the inequality is:

12 ≤ 6 · x ≤ 36

b) And the solutions for the values of x are represented by the following inequality:

2 ≤ x ≤ 6

To learn more on inequalities: https://brainly.com/question/28823603

#SPJ1

help me out with this question..this is a k11..remember this is a practice question not a graded one

Answers

Each flight has a probability of 60% or 0.6 of being on time. This means that its complement, or the probability that the flight isn't on time is:

[tex]\text{\textasciitilde{}P(on time)}=1-0.6=0.4[/tex]

It is 40% or 0.4. "~P(on time)" stands for the probability of the flight not being on time.

1. The probability that at least 2 flights are on time is:

To find the probability that 2 or more flights are on time we can fight the probability that "0" or "1" are not on time.

[tex]P(0\text{ on time)}=0.4^9=0.000262144[/tex][tex]\begin{gathered} P(1\text{ on time})=\frac{9!}{1!\cdot(9-1)!}\cdot0.6\cdot(0.4)^8 \\ P(1\text{ on time)}=9\cdot0.6\cdot(0.4)^8=0.003538944 \end{gathered}[/tex][tex]\begin{gathered} P(1\text{ or less on time)}=P(0\text{ on time)}+P(1\text{ on time)} \\ P(1\text{ or less on time)}=0.000262144+0.003538944=0.003801088 \end{gathered}[/tex]

The probability of 2 or more flights are on time is:

[tex]P(2\text{ or more on time)}\cdot=1-0.003801088=0.996198912[/tex]

The probability of 2 flights or more are on time is 0.996198912

2.

We need to calculate the probabilities of 7,8 and 9 flights are on time and then subtract by 1.

[tex]\begin{gathered} P(7)=\frac{9!}{7!\cdot(9-7)!}\cdot0.6^7\cdot0.4^2 \\ P(7)=36\cdot0.6^7\cdot0.4^2=0.161243136 \end{gathered}[/tex][tex]\begin{gathered} P(8)=\frac{9!}{8!\cdot(9-8)!}\cdot0.6^8\cdot0.4 \\ P(8)=9\cdot0.6^8\cdot0.4=0.060466176 \end{gathered}[/tex][tex]P(9)=0.6^9=0.010077696[/tex]

The probability of at most 6 flights are on time is:

[tex]\begin{gathered} P(6\text{ or less on time) = 1 - (}P(7)+P(8)+P(9)) \\ P(6\text{ or less on time) = 1-(0.161243136+0.060466176+0.010077696)=}0.768212992 \end{gathered}[/tex]

The probability of 6 or less are on time is 0.768212992.

3.

The probability of exactly 5 flights are on time is:

[tex]\begin{gathered} P(5)=\frac{9!}{5!(9-5)!}0.6^5\cdot0.4^4 \\ P(5)=126\cdot0.6^5\cdot0.4^4=0.250822656 \end{gathered}[/tex]

The probability of exactly 5 flights are on time is 0.250822656.

Solve for z in -3 < z-1 < 3.Give the result in the interval notation and graph on a number line

Answers

Answer:

(-2,4)

Explanation:

Given the inequality:

[tex]-3First, we add 1 to all parts of the inequality.[tex]\begin{gathered} -3+1We can represent this in interval notation as:[tex](-2,4)[/tex]

The solution set is graphed on the number line below:

an arithmetic sequence has these properties: a1=2an = an-1 + 5 what are the first four terms of the sequence?

Answers

we have that the arithmetic sequence have:

[tex]\begin{gathered} a_1=2 \\ a_n=a_{n-1}+5 \end{gathered}[/tex]

so we can replace for n=2,3,and 4 to find the first four terms so:

for n=2

[tex]\begin{gathered} a_2=a_{2-1}+5 \\ a_2=a_1+5 \\ a_2=2+5 \\ a_2=7 \end{gathered}[/tex]

for n=3

[tex]\begin{gathered} a_3=a_{3-1}+5 \\ a_3=a_2+5 \\ a_3=7+5 \\ a_3=12 \end{gathered}[/tex]

and for n=4

[tex]\begin{gathered} a_4=a_{4-1}+5 \\ a_4=a_3+5 \\ a_4=12+5 \\ a_4=17 \end{gathered}[/tex]

So the first four numbers are

[tex]2,7,12,17[/tex]

I’m confused because I don’t know how to answer this question

Answers

SOLUTION:

We are to find the domain of the given exponential function shown;

The domain of exponential functions is all real numbers.

So the correct option is D

Brian had p pencils. Then he bought 4 more.
How many does he have now? due tm pleaseeeeeeeeeeeeee help

Answers

Answer: The number of pencils Brian has is p+4

Step-by-step explanation:

Because p is the original number of the pencil ( we don't know how many are there so we keep it p). Then he bought 4 more which mean +4, so the answer is p+4

point(10) On Myra's family vacation, her mom drove on the highway at aconstant speed of 60 miles per hour. Based on this rate, which of thefollowing times and miles driven are correct? Select all that apply. GRP.30190 miles driven in 3 hours150 miles driven in 2.5 hours30 miles driven in 30 minutes

Answers

[tex]\begin{gathered} \text{speed}=\frac{dis\tan ce}{\text{time}} \\ \text{speed}=60\text{ miles per hour} \\ \text{The possible speed = }\frac{150}{2.5}=60\text{ miles per hour} \\ 30\text{ miles per 30 minutes is also correct} \end{gathered}[/tex]

the product of 5 - 2i and i is

Answers

Answer:

5i + 2

Step-by-step explanation:

(5-2i)(i)

5i - 2i^2

i^2= -1

5i -2(-1)

= 5i + 2

The expression -7y is a ____. (20 Points!!) (Please help!)

Term
Constant
Variable

Answers

Answer:

constant

Step-by-step explanation:

-7y is a contant

6. Find the are of the Rectangle
776
in
13
in

Answers

Answer:

about 3.09

Step-by-step explanation:

A = l  x w

A = 13/7 x 7/6

A = 1.857 x 1.666

A = 3.09

Hope it helps!

There are 27 students in Mrs. Mello's class. fin the total number of pages they read by the end of November.

Answers

The total number of pages they read by the end of November is 2916 pages.

As the question, Chapter 1 has 35 pages, Chapter 2 has 38 pages and Chapter 3 has 35 pages. All these pages are read by all 27 students by the end of November.

⇒   Total number of pages in Three chapters  [tex]=35+38+35[/tex]

                                                                             [tex]=108[/tex]

⇒   Total number of pages in all three chapters read by one student = 108

⇒   Total number of pages read by 27 students  

                                      [tex]= total number of pages in three chapters * 27[/tex]

                                      [tex]=108*27[/tex]

                                      [tex]=2916[/tex]

Therefore, The total number of pages 27 students read by the end of November = 2916.

To know more about pages problems:

https://brainly.com/question/25996348

The full question is

There are 27 students in Mrs. Mello's class. fin the total number of pages they read by the end of November.

           

                                                   

ſ x - 5y = -9( 4x + 4y = - 12Step 1 of 2: Determine if the point (-4,1) lies on both of the lines in the system of equations by substituting theordered pair into both equations.

Answers

Given:

[tex]\begin{gathered} x-5y=-9\ldots\ldots(1) \\ 4x+4y=-12\ldots\text{.}\mathrm{}(2) \end{gathered}[/tex]

The given point is (-4,1)

Let's check the ordered pair (-4,1) in the first equation

[tex]\begin{gathered} x-5y=-9 \\ (-4)-5(1)=-9 \\ -4-5=-9 \\ -9=-9 \end{gathered}[/tex]

Hence, (-4,1) is a solution of the first equation x-5y=-9.

Now, let's check (-4,1) in the second equation.

[tex]\begin{gathered} 4x+4y=-12 \\ 4(-4)+4(1)=-12 \\ -16+4=-12 \\ -12=-12 \end{gathered}[/tex]

So, (-4,1) is a solution of the second equation 4x+4y=-12.

Hence, (-4,1) is a solution of both equation in the system, then it is a solution to the overall system.

Other Questions
How long should you wait before sending a thank-you letter following aninterview? A. One weekB. One day C. One monthD. Two to three days Find the missing the side of the triangle.A. 30 ydB. 17 ydC. 25 ydD. 0 yd Hey can you help me with my homework also can you tell me the points so I can put them into the graphs Given vector v equals open angled bracket negative 11 comma negative 5 close angled bracket comma what are the magnitude and direction of v? Round the magnitude to the thousandths place and the direction to the nearest degree. solve for 18 degreex 29 The glass portion of a small window is 12 inches by 24 inches. The framework on each side adds on x inches. Express the area of the entire window as a function of x. 72 + x square inches 36+x square inches x + 36x + 288 square inches 288 + x square inches What are the solution(s) to the quadratic equation 9x = 4?O x = and x = -- 90x = and x = -1/33O= and x = --X=no real solutionM/NK what would the length of segment BC have to be in order for line BC to be tangent to circle PLEASE HELP AS SOON AS POSSIBLE 3(x + 10) < 2 (20 x) give some examples in mathematics for a 4th grader. 3. When you cut your skin, the body secretes platelets to help form a clot. Theclot formation continues until the wound is sealed.ChoosePositive feedbackNegative feedback cit Formula = 18 + 7(58 - 1) 58 CRIBE EN ORACIONES COMPLETAS What is 2902 divided by 3 Find the area of the region enclosed by y = 7x and y = 8x^2. A straight line passes through points (1, 15) and(5, 3) What isthe equation of the line?Select one:A) y = - 3x + 18B) y = 7x + 18C) y = 2x + 18D) y = 3x + 18 A bag contains the following marbles: 12 black marbles, 8 blue marbles, 16 brown marbles and 14 green marbles. what is the ratio of black marbles to blue marbles. If _____________, then the graph of the polynomial function is symmetric about the origin.f(x) = -f(-x)f(x) = -f(x)f(x) = f(-x)f(x) = f(x + 1) A junk drawer at home contains eight pens four of which work what is the probability that a randomly grab three pens from the drawer and dont end up with a pen that works express your answer as a fraction in lowest terms or decimal rounded to the nearest million pls answer question will mark brainliset tyty